Lagrange-Funktion eines Teilchens im elektromagnetischen Feld?

2 Antworten

Vom Fragesteller als hilfreich ausgezeichnet

Ich habe dir mal einen kleinen Aufschrieb hinterlassen. Dieser sollte helfen, denn es zeigt, wie man aus der Lorentzkraft F das Potential V erhält, um die Lagrangefunktion zu bilden, die du ja suchst. Zudem erklärt es, woher der Term in der Lagrangefunktion kam, der unklar war

Bild zum Beitrag

Bild zum Beitrag

Woher ich das weiß:Berufserfahrung – Theoretischer Physiker (Vielteilchensysteme, Quantenoptik)
 - (Physik, Physikstudium, Mechanik Physik)  - (Physik, Physikstudium, Mechanik Physik)

daraus alleine bekommst du die Lagrange funktion eh nicht.

aber du kannst den umgekehrten weg gehen und aus der Lagrange funktion die bewegungsgleichung herleiten und zeigen dass sie der Lorentz-kraft entspricht.

Woher ich das weiß:Berufserfahrung – Physiker (Teilchenphysik)

andi3366 
Fragesteller
 22.02.2023, 18:12

Angenommen man würde die Lorentzkraft nicht kennen. Wie geht man dann vor, um aus den Maxwellschen Gleichungen die Lagrangefunktion zu bestimmen und daraus den Ausdruck der Lorentzkraft zu erhalten?

0
Reggid  22.02.2023, 20:23
@andi3366

naja, du könntest zeigen dass die Lagrange-dichte die Maxwell-gleichungen für das potential und stromdichte eines punktteilchens als bewegungsgleichung liefert wenn du nach phi und A variierst.

das geht aber wesentlich einfacher in kovarianter formulierung. weiß aber nicht ob ihr das gemacht habt.

0
iSolveProblems  22.02.2023, 20:46
@Reggid

Das macht man eigentlich so richtig ordentlich in einer SRT/ART Vorlesung oder einer guten Elektrodynamik am Ende

0
iSolveProblems  22.02.2023, 21:59
@Reggid

Ne, da macht man elektro und magnetostatik? Wieso sollte man dann relativistisch rechnen? 😂

0
iSolveProblems  22.02.2023, 21:08

Ach und apropos lorentz Kraft. Ich habe den Ansatz jetzt so gemacht , dass ich die Kraft Benutze um das Potential abzuleiten und die lagrangefunktion über T-V auszudrücken

0